You are on page 1of 53

CIRCUIT I

Introduction to Electric Circuit


Electric Charge - when there’s a flow of charge, there’s a flow of electricity.
- when there’s a movement of electron from one body to another, there’s a
flow of charge where electricity is created.
Charge - is an electrical property of the atomic particles of which matter consists, measured in
coulombs (C).
- a body is said to be charged, if it has either an excess or deficit of electrons from its
normal values due to sharing

• 1 coulomb = 6.25 × 1018 𝑒 −


• 1 electron = 1.6 × 10−19 𝐶
Electric Current - time rate of change of charge, measured in amperes (A).
- the relationship between current i, charge q and time t are:
𝑑𝑞 𝑞
i= ,I =
𝑑𝑡 𝑡
- current is measured in amperes, where 1 ampere = 1 coulomb/sec
Ampere – unit of charge flow equal to one coulomb of charge past a given point in one
second. Named after Andre M. Ampere (1775 – 1836)
Alternating Current – current that varies sinusoidally with time
Direct Current – current that remains constant with time.
Sample Problems
1.) The total charge entering a terminal is given by 𝑞 = 5𝑡 𝑠𝑖𝑛4𝜋𝑡 𝑚𝐶. Calculate the current at t
= 0.5 s
Given:
𝑞 = 5𝑡 𝑠𝑖𝑛4𝜋𝑡 𝑚𝐶 𝑢𝑣 = 𝑢𝑑𝑣 + 𝑣𝑑𝑢 𝑠𝑖𝑛𝑥 = 𝑐𝑜𝑠𝑥𝑑𝑥
t = 0.5 s
𝑑𝑞 𝑢 = 5𝑡 𝑣 = 𝑠𝑖𝑛4𝜋𝑡 = 𝑐𝑜𝑠4𝜋𝑡(4𝜋)
i=
𝑑𝑡 𝑑𝑢 = 5 𝑑𝑣 = 4𝜋𝑐𝑜𝑠4𝜋𝑡
𝑑(5𝑡 𝑠𝑖𝑛4𝜋𝑡 ) = 4𝜋𝑐𝑜𝑠4𝜋𝑡
i=
𝑑𝑡

𝑖 = 5𝑡(4𝜋𝑐𝑜𝑠4𝜋𝑡) + 𝑠𝑖𝑛4𝜋𝑡(5)

= 20𝜋𝑡𝑐𝑜𝑠4𝜋𝑡 + 5𝑠𝑖𝑛4𝜋𝑡

= 20𝜋(0.5)𝑐𝑜𝑠4𝜋(0.5) + 5𝑠𝑖𝑛4𝜋(0.5)

= 10𝜋𝑐𝑜𝑠2𝜋 + 5𝑠𝑖𝑛2𝜋

= 10𝜋(1) + 5(0)

𝑖 = 10𝜋 𝐴
2.) The total charge entering a terminal of a certain dielectric plate is given by the equation
𝑞 = (10 − 10𝑒 −2𝑡 ). Calculate the value of current at elapsed time t = 0.5 s.

Given: 𝑖 = 0 − [10𝑒 −2𝑡 (−2)]


−2𝑡
𝑞 = 10 − 10𝑒 = 20𝑒 −2𝑡
t = 0.5 s
𝑑𝑞 = 20𝑒 −2(0.5)
𝑖=
𝑑𝑡 = 20𝑒 −1
𝑑(10 − 10𝑒 −2𝑡
)
𝑖= 20
𝑑𝑡 𝑒
𝑖=
𝑜𝑟 7.3575 𝐴
3.) Determine the total charge entering a terminal between t = 1s and t = 2s if the current
passing the terminal is 𝑖 = 3𝑡 2 − 𝑡 𝐴.
2 1
Given: 𝑞 = ∫ (3𝑡 2 − 𝑡)𝑑𝑡 = [8 − 2] − [1 − 2]
1
𝑡1 = 1s 1
3𝑡 3 𝑡 2 =6−2
𝑡2 = 2s = −
3 2 = 5.5 𝐶
𝑖 = 3𝑡 2 − 𝑡 2
𝑡
= 𝑡3 −
2
𝑞 = ∫ 𝑖𝑑𝑡
(2)2 (1)2
= [(2)3 − ] − [ (1)3 − ]
2 2

4.) The current in an electric lamp is five amperes. What quantity of electricity flow towards the
filament in six minutes?
Given: 60 𝑠𝑒𝑐
𝑡 = 6 𝑚𝑖𝑛 × = 360𝑠
1 𝑚𝑖𝑛
i = 5A
𝑞
t = 6 mins 𝑖=
𝑡
Charge = C 𝑞 = 𝑖𝑡
Current = A 𝑞 = (5𝐴)(360𝑠)
Time = 6 𝑞 = 1800 𝐶

Protons – positively charge particles that stays in the nucleus of an atom.

– mass of 1 proton is 1.673 × 10−27 𝑘𝑔

Neutrons – particles having no charge


– have the same mass as protons but their mass is slightly greater,

1.675 × 10−27 𝑘𝑔

Electrons – negatively charged particles that revolves around the nucleus of an atom.

– one of the lightest particles, the mass of an electron is about 9.11 × 10−31 𝑘𝑔
Total Number of Electrons on a Given Shell

𝑁 = 2𝑛2 𝑛 = 𝑛𝑡ℎ 𝑠ℎ𝑒𝑙𝑙 𝑜𝑓 𝑎𝑛 𝑎𝑡𝑜𝑚


𝑁 = 𝑡𝑜𝑡𝑎𝑙 𝑛𝑢𝑚𝑏𝑒𝑟 𝑜𝑓 𝑒𝑙𝑒𝑐𝑡𝑟𝑜𝑛𝑠 𝑜𝑛 𝑎 𝑔𝑖𝑣𝑒𝑛 𝑠ℎ𝑒𝑙𝑙
Sample Problem:
1.) Determine the maximum number of electrons that can be existed in the 3 rd shell of an atom.
𝑁 = 2𝑛2

= 2(3)2

= 2(9)

𝑁 = 18 𝑒𝑙𝑒𝑐𝑡𝑟𝑜𝑛𝑠

2.) If the given shell contains 32 number of electrons, locate the position of the given shell.

𝑁 = 2𝑛2

𝑁
𝑛 = √2

32
=√
2

= √16

𝑛 = 4𝑡ℎ 𝑠ℎ𝑒𝑙𝑙

Conductors – materials having less than four valence electrons

Semi-Conductors – materials having four valence electrons

Insulators – materials having more than four valence electrons

Sample Problems:

1.) How much charge is represented by 4600 electrons?

1.6 × 10−19 𝐶
4600𝑒 − × = 7.36 × 10−16 𝐶
1𝑒 −
2.) A dielectric contains 13.67 × 1021 free electrons. How much charge in coulomb the dielectric
will generate?

1.6 × 10−19 𝐶
13.67 × 1021 × = 2187.2 𝐶
1𝑒 −
3.) A positive charged dielectric has a charge of 2 coulombs. If 12 × 1018 free electrons are added
to it, what will be the charge on the said dielectric?
1.6 × 10−19 𝐶
𝑞𝑇 = 𝑞1 + 𝑞2 𝑞2 = 12 × 1018 × = −1.92 𝐶
1𝑒 −
𝑞2 = +2𝐶 𝑞𝑇 = +2𝐶 + (−1.92𝐶 ) = 0.08 𝐶
Potential – the capability of doing work

• to move the electron in a conductor in a particular direction requires some work or


energy transfer. This work is performed by an external electromotive force (emf),
typically represented by the battery.
• any charge has the capability of doing work by moving another charge either by
repulsion or attraction.

Volt – unit potential difference which equal to one joule of work done per one coulomb of charge.
Named after Alessandro C. Volta (1754-1827), inventor of the first battery.

1 volt = 1 joule/coulomb = 1 newton meter/coulomb


𝑑𝑤 𝑤
𝑉𝐴𝐵 = , 𝑉=
𝑑𝑞 𝑞

Power – is the time rate of expanding or absorbing energy, measured in watts.

– power p is a time-varying quantity and is called instantaneous power. Thus, the power
absorbed or supplied by an element is the product of the voltage across the element and the
current through it.
𝑑𝑤 𝑤
𝑃= , 𝑃 = 𝑖𝑣 , 𝑃=
𝑑𝑡 𝑡
Energy – is the capacity to do work, measured in joules (J)

– electric power utility companies measure energy in watt-hours (Wh)

1 Wh = 3600J

Sample Problem:

1.) An energy source forces a constant current of 2A for 10s to flow through a light bulb. If 2.3
kJ is given off in the form of light and heat energy, calculate the voltage drop across the bulb.

Given: 𝑞 = 𝐼𝑡 𝑤
𝑤 𝑉=
𝑉= 𝑞
I = 2A 𝑞 = (2𝐴)(10𝑠)
𝑤 2.3 × 103 𝐽
t = 10s =
𝑃= 𝑞 = 𝐼𝑡 𝑞 = 20 𝐶 20 𝐶
𝑡
𝐸
= 2.3𝑘𝐽 𝑃 𝑉 = 115 𝑉
𝑤
𝑃 = 𝐼𝑉 𝑉=
𝐼

2.) A battery can deliver 10 joules of energy to move 5 coulombs of charge. What is the
potential difference between the terminals of the battery?
𝑤
𝑉=
𝑞
10 𝐽
=
5𝐶
𝑉 = 2𝑉
3.) To move charge q from point a to point b requires -30J. Find the voltage drop 𝑉𝐴𝐵 if: (a)
q = 2 C, (b) q = -6 C
𝑤 𝑤
a.) 𝑉 = b.) 𝑉 =
𝑞 𝑞

−30 𝐽 −30 𝐽
= =
2𝐶 −6 𝐶
𝑉 = −15𝑉 𝑉 = 5𝑉

4.) How much energy does a 100W electric bulb consume in two hours?
𝑤
𝑃= 𝐸 = 𝑃𝑡
𝑡
𝐸
𝑃= = 100𝑊
𝑡
𝐸 = 200 𝑊ℎ𝑟

5.) Assume that the voltage at the terminals of the element whose current is 20𝑒 −5000𝑡 A
are:

V = 0V t < 0;

V = 10𝑒 −5000𝑡 t>0

a) Calculate the power supplied to the element at 1 ms


b) Calculate the total energy (in joules) delivered to the circuit element
1 ℎ𝑜𝑢𝑟 𝑤
1 × 10−3 s × = 2.78 × 10−7 hours 𝑝=
3600 𝑠𝑒𝑐 𝑡
𝑃 = 𝐼𝑉 𝐸
=
𝑡
= (20𝑒 −5000𝑡 )(10𝑒 −5000𝑡 )
𝐸 = 𝑃𝑡
= 200𝑒 −5000𝑡
−5000(1×10−3 )
= (9.08 × 10−3 )(2.78 × 10−7)
= 200𝑒
= 2.52 × 10−9 𝑊ℎ𝑟
= 200𝑒 −10
3600 𝐽
𝑃 = 9.08 × 10−3 𝑊𝑎𝑡𝑡𝑠 𝐸 = 2.52 × 10−9 × = 9.09 × 10−6 J
1 𝑊ℎ𝑟
Electrical Resistance – the resistance (R) of an element denotes its ability to resist the flow
of electric current, measured in Ohms.

– Ohms (Ω) practical unit of resistance, named after the German


Physicist George S. Ohm (1787-1854).

Law of Resistance – varies directly as its length

– varies inversely as the cross sectional are of the conductor

– depends on the nature of the material

– depends on the temperature of the conductor


𝑝𝐿 𝑝𝑉 𝑝𝐿2 𝑝𝐿 𝐴 𝑝𝑉
𝑅= , 𝑅= , 𝑅= 𝑅= × =
𝐴 𝐴2 𝑉 𝐴 𝐴 𝐴2
Where: R = resistance in Ohms 𝑝𝐿 𝐿 𝑝𝐿2
𝑅= × =
A = cross-sectional area 𝐴 𝐿 𝑉

P = resistivity (ohm-meter)

Specific Resistance (resistivity) – resistance offered by a unit cube of the material

Circular Mil (CM) – area of the circle having a diameter of one mil

CM = 𝑑 2 , where d is the diameter of the given material in terms of Mil

1000 mil = 1 inch

1 MCM = 1000 CM

Resistivity of Materials (p)

Material Resistivity (Ω - CM/ft)


Silver 9.9
Copper 10.37
Aluminum 17
Tungsten 33
Zinc 36

Sample Problems

1.) A copper conductor has its diameter of 0.12 inch. Compute its cross-sectional area in
terms of circular mil.

Given: 𝐴 = 𝑑2

1000 𝑚𝑖𝑙 = (120)2


0.12 inch × = 120 𝑚𝑖𝑙
1 𝑖𝑛𝑐ℎ
𝐴 = 14400

2.) The substation bus bar is made up of 2 inches round copper bars 20 ft long. What is
the resistance of each bar if the resistivity is 10.37 ohm-CM/ft?

Given: 𝐴 = 𝑑2 𝑝𝐿
𝑅=
d = 2 inch 𝐴
= (120)2
(10.37 Ω − CM/ft)(20𝑓𝑡)
L = 20 ft =
= 4 × 106 𝐶𝑀 4 × 106 𝐶𝑀
P = 10.37 Ω-CM/ft
= 5.185 × 10−5 Ω
1000 𝑚𝑖𝑙
𝑑 = 2 inch × = 2000 𝑚𝑖𝑙 𝑅 = 51.85 𝜇Ω
1 𝑖𝑛𝑐ℎ
3.) A coil consists of 2000 turns of copper wire having cross-sectional area of 0.8 𝑚𝑚2 .
The mean length per turn is 80 cm and the resistivity of the copper is 00.02 micro-ohm
meter. Find the resistance if the coil.

Given: 𝐿 = 𝑁𝐼 1𝑚
𝐴 = 0.8 𝑚𝑚2 × ( )2
1000 𝑚𝑚
N = 2000 turns = (2000)(80)
= 8 × 10−7 𝑚2
2
A = 0.8 𝑚𝑚 = 160,000 𝑐𝑚
𝑝𝐿
l = 80 cm/turn 1𝑚
𝑅=
= 160,000 𝑐𝑚 × 100 𝑐𝑚 = 1600 𝑚 𝐴
p = 0.02 μΩ× 𝑚 (0.02 × 10−6 )(1600)
=
8 × 10−7
𝑅 = 40Ω

With the volume of the material being constant, resistance varies directly as the square of
the length.

𝑝𝐿2 𝑝1 𝐿12 𝑝2 𝐿22 𝑅1 = 𝐾1 𝐿12 𝑅2 = 𝐾2 𝐿22


𝑅= , 𝑅1 = , 𝑅2 =
𝑉 𝑉1 𝑉2 𝐾1 = 𝐾2
𝐿1 ≠ 𝐿2 𝑅 𝑅
𝐾1 = 𝐾2 =
𝐿12 𝐿22
𝑑1 ≠ 𝑑2
𝑅1 𝑅2
𝐾1 − 𝑃1 = 𝑃2 2 =
𝐿1 𝐿22
𝐾2 − 𝑉1 = 𝑉2
𝑅1 𝐿12
=
𝑅2 𝐿22
𝑅1 𝐿1
= [ ]2
𝑅2 𝐿2

With the volume of the material constant, resistance varies inversely as to the fourth power
of the diameter.
𝑝𝑉 𝑝1 𝑉1 𝑝2 𝑉2 𝐾1 𝐾2
𝑅= , 𝑅1 = , 𝑅2 = 𝑅1 = 𝑅1 =
𝐴2 𝐴12 𝐴22 𝐴12 𝐴22

𝑝1 𝑉1 = 𝐾1 𝑝2 𝑉2 = 𝐾2 𝐾1 𝐾2
𝑅1 = 𝑅1 =
(𝑑12 )2 (𝑑22 )2
𝐴 = 𝑑2
𝐾1 𝐾2
𝐾1 = 𝐾2 𝑅1 = 𝑅1 =
𝑑14 𝑑24

𝐾1 = 𝑅1 𝑑14 𝐾2 = 𝑅2 𝑑24

𝑅1 𝑑14 = 𝑅2 𝑑24

𝑅1 𝑑24
=
𝑅2 𝑑14

𝑅1 𝑑2
= [ ]4
𝑅2 𝑑1
Sample Problems
1.) A one-meter rod of 2 cm diameter is drawn until its resistance is 100 times the initial
resistance. Its length afterward is?
𝑅1 𝐿1
Given: = [ ]2
𝑅2 𝐿2
𝐿1 = 1 𝑚 𝑅1 1𝑚
= [ ]2
𝑑1 = 2 𝑐𝑚 100𝑅1 𝐿2

𝑅2 = 100𝑅1 1 1𝑚2
= 2
100 𝐿2

𝐿2 2 = 100𝑚2

𝐿2 = √100𝑚2

𝐿2 = 10𝑚
2.) A kilometer of wire having a diameter of 11.7 mm and a resistance of 0.031Ω is drawn
down so that its diameter is 5.0 mm. What does its resistance become?
Given: 𝑅1 𝑑14 = 𝑅2 𝑑24
𝐿1 = 1 𝑘𝑚 (0.031Ω)(11.77𝑚𝑚)4 = 𝑅2 (5.00𝑚𝑚)4
𝑑1 = 11.7 𝑚𝑚 (0.031Ω)(11.77𝑚𝑚)4
𝑅2 =
𝑑2 = 5 𝑚𝑚 (5.0𝑚𝑚)4

𝑅1 = 0.031Ω 𝑅2 = 0.93 Ω

3.) A certain wire has a resistance R. The resistance of another wire identical with the first
except having twice its diameter is…
𝑃1 = 𝑃2 𝑅1 𝑑2
Given:
= [ ]2
𝐿1 = 𝐿2 𝑅2 𝑑1
𝑑2 = 2𝑑1
𝑑1 ≠ 𝑑2 𝑅1 2𝑑1 2
=[ ]
𝑅2 𝑑1
𝑉1 = 𝑉2
4𝑑1 2
𝑝1 𝐿1 𝑝2 𝐿2 =
𝑅1 = 𝑅2 = 𝑑1 2
𝐴1 𝐴2
𝑅1
𝐾1 𝐾2 =4
𝑅1 = 𝑅2 = 𝑅2
𝐴1 𝐴2
𝑅1
𝐴 = 𝑑2 𝑅2 =
4
𝐾1 𝐾2
𝑅1 = 𝑅2 =
𝑑12 𝑑22

𝐾1 = 𝐾2

𝑅1 𝑑12 = 𝑅2 𝑑22
𝑅1 𝑑2
= [ ]2
𝑅2 𝑑1
The resistance of all wires increases as the temperature increases
𝑅2 𝑅1 = initial resistance
= 1 + 𝛼𝑡1 △ 𝑇
𝑅1 𝑅2 = final resistance

𝑅2 𝑇 + 𝑡1 𝑇 = inferred absolute temperature


=
𝑅1 𝑇 + 𝑡2
= temperature when resistance of a given materials is zero
1
𝛼𝑡1 = 𝑡1 = initial temperature
𝑇 + 𝑡1
𝑡2 = final temperature
△ 𝑇 = 𝑡2 − 𝑡1
△ 𝑇 = change in temperature

𝛼 = temperature coefficient of resistance


Sample Problems

1.) A coil copper wire has a resistance of 100 when its temperature is 0°C. Determine its
resistance at 70°C if the temperature coefficient of resistance of copper at 0°C is 0.0043/°C.

Given: 𝑅2 = 𝑅1 (1 + 𝛼𝑡1 △ 𝑇)
𝑅1 = 100Ω = 100Ω[1 + 0.0043/°C(70°C − 0°C)]
𝑡1 = 0°C 𝑅2 = 130.1Ω
𝑡2 = 70°C

𝛼𝑡1 = 0.0043/°C

2.) A coil of copper wire has a resistance of 10Ω at 20°C. If the temperature coefficient of
resistance of copper at 20°C is 0.004/°C. Determine the resistance of the coil when the
temperature rises to 100°C.

Given: 𝑅2 = 𝑅1 (1 + 𝛼𝑡1 △ 𝑇)
𝑅1 = 10Ω = 10Ω[1 + 0.004/°C(100°C − 20°C)]
𝑡1 = 20°C 𝑅2 = 13.2Ω
𝑡2 = 100°C

𝛼𝑡1 = 0.004/°C

3.) An aluminum cable has a resistance of 27Ω at a temperature of 35°C. Determine its
resistance at 0°C. Take the temperature coefficient of resistance at 0°C to be 0.0038/°C.

Given: 𝑅2 = 𝑅1 (1 + 𝛼𝑡1 △ 𝑇)
𝑅2 = 27Ω 𝑅2
𝑅1 =
𝑡1 = 0°C 1 + 𝛼𝑡1 △ 𝑇

𝑡2 = 35°C 27Ω
=
1 + 0.0038/°C(35°C − 0°C)
𝛼𝑡1 = 0.0038/°C
𝑅1 = 23.82Ω
4.) A carbon resistor has a resistance of 1 kΩ at 0°C. Determine its resistance at 80°C. Assume
that the temperature coefficient of resistance for carbon at 0°C is -0.0005/°C.

Given: 𝑅2 = 𝑅1 (1 + 𝛼𝑡1 △ 𝑇)
𝑅1 = 1000Ω = 1000Ω[1 + (−0.0005/°C)(80°C − 0°C)]
𝑡1 = 0°C 𝑅2 = 960Ω
𝑡2 = 80°C

𝛼𝑡1 = −0.0005/°C

Ohm’s Law states that the current flowing in an electric circuit is directly proportional to the
impressed voltage applied to the circuit and inversely to the equivalent resistance of the
said circuit.

Electrical Power – power is the rate of energy transfer

– watt, unit of electrical energy equal to one joule of energy consumed in


one second. Named after James Watt (1736-1819).

𝑉 𝑉2
𝑃 = 𝐼𝑉 𝐼= 𝑉 = 𝐼𝑅 𝑃 = 𝐼𝑉 = 𝐼 2 𝑅 = P = electrical power
𝑅 𝑅
V = voltage
𝑉2
𝑃= 𝑃 = 𝐼𝑉
𝑅 I = current
= 𝐼(𝐼𝑅) R = resistance
𝑃 = 𝐼2𝑅

Sample Problem

1.) A 200 V lamp has a hot resistance of 400Ω. The power rating in watts of the lamp is?

Given: 𝑉2
𝑃=
𝑅
V = 200 V
(200)2
R = 400Ω =
400Ω
𝑃 = 100𝑊
2.) A 5 Hp motor drives a mechanical load, taking 18.8 A from a 230 V source. Calculate
the power input of the motor.

Given: 𝑃 = 𝐼𝑉
V = 230 V = (18.8 A)(230 𝑉)
I = 18.8 A 𝑃 = 4324𝑊
Energy – is defined as the capacity to do work
Gram-calorie or calorie – the energy required to raise the temperature of 1 gram of water by
1°C
British Thermal Unit (BTU) – the energy required to raise the temperature of 1 pound of
water by 1°F
Electrical and Heat Energy Equivalent Formula
𝐸 = 𝑃𝑡 𝐸 = electrical energy (joules) 1 𝑐𝑎𝑙𝑜𝑟𝑖𝑒 = 4.186 𝐽

𝑄 = 𝑚𝐶 △ 𝑇 𝑄 = heat energy 1 𝐵𝑇𝑈 = 252 𝑐𝑎𝑙𝑜𝑟𝑖𝑒𝑠

𝑃 = electrical power 1 𝑗𝑜𝑢𝑙𝑒 = 1 × 107 𝑒𝑟𝑔𝑠

𝐶 = specific heat of the material 1 𝑘𝑊ℎ = 3600 𝐽 = 3413 𝐵𝑇𝑈 = 860 𝑘𝐶𝑎𝑙

𝑡 = time (seconds)

𝑚 = mass of the material

△ 𝑇 = change in temperature
Specific Heat of Water: 1 cal/gram •°C
1 kcal/kg •°C
4186 J/kg •°C
1 BTU/lb •°F
Sample Problem
1.) A residential house has a lightning load of 1000W and a small appliance load of 2000W.
If they are used at the same time, what will be the monthly bill at an energy cost of ₱0.40
per kilowatt-hour? 𝑃 =𝑃 +𝑃𝑇 1 2
Given: = 1000𝑊 + 2000𝑊
𝑃1 = 1000𝑊
= 3000𝑊 𝑜𝑟 3𝑘𝑊
𝑃2 = 2000𝑊
𝐸 = 𝑃𝑡
𝑟𝑎𝑡𝑒 = 0.40/𝑘𝑊ℎ
= 3𝑘𝑊(720 ℎ𝑜𝑢𝑟𝑠)
𝑡 = 1 𝑚𝑜𝑛𝑡ℎ
= 2160 𝑘𝑊ℎ
30 𝑑𝑎𝑦𝑠 24 ℎ𝑜𝑢𝑟𝑠
= 1 𝑚𝑜𝑛𝑡ℎ × × = 720 ℎ𝑜𝑢𝑟𝑠 𝐵𝑖𝑙𝑙 = 𝐸 (𝑟𝑎𝑡𝑒)
1 𝑚𝑜𝑛𝑡ℎ 1 𝑑𝑎𝑦
= 2160 𝑘𝑊ℎ(0.40)
𝐵𝑖𝑙𝑙 = ₱864

2.) 100 liters of water is heated from 20°C to 40°C. How many kWh of electricity is needed
assuming no losses?
Given: 𝐸=𝑄 1𝑘𝑔 𝐸 = 𝑚𝐶 △ 𝑇
𝑃𝑊 = 1𝑘𝑔 − 1𝐿
𝑇1 = 20°C 𝐿
𝑄 = 𝑚𝐶 △ 𝑇 = (100𝑘𝑔)(1 𝑘𝑐𝑎𝑙/𝑘𝑔 • °C)(40°C − 20°C)
𝑚
𝑇2 = 40°C =
𝐸 = 𝑚𝐶 △ 𝑇 𝑣 = 2000 kcal

𝑉 = 100𝐿 1𝑘𝑔 1𝑘𝑊ℎ


𝑚= (100𝐿) = 2000 kcal ×
𝐿 860 𝑘𝑐𝑎𝑙
𝐶 = 1 𝑘𝑐𝑎𝑙/𝑘𝑔 • °C
= 100 𝑘𝑔 𝐸 = 2.33 𝑘𝑊ℎ
Series Circuit
The total resistance is equal to the sum of the individual resistances

𝐼𝑇 = 𝐼1 = 𝐼2

𝑉𝑇 = 𝑉1 = 𝑉2
𝑉
𝐼= 𝑉1 = 𝐼1 𝑅1
𝑅
Voltage Divider Equation 𝑉 = 𝐼𝑅
𝐼𝑇 = 𝐼1 𝐼𝑇 = 𝐼2
𝐼𝑇 𝑅𝑇 = 𝐼1 𝑅1 + 𝐼2 𝑅2
𝑉 𝑉
𝐼= 𝐼= 𝐼𝑇 𝑅𝑇 = 𝐼𝑇 𝑅1 + 𝐼𝑇 𝑅2
𝑅 𝑅
𝑉𝑇 𝑉1 𝑉𝑇 𝑉2 𝐼𝑇 𝑅𝑇 = 𝐼𝑇 (𝑅1 + 𝑅2 )
= =
𝑅𝑇 𝑅1 𝑅𝑇 𝑅2 𝑅𝑇 = 𝑅1 + 𝑅2 … + 𝑅𝑁
𝑉𝑇 𝑅1 𝑉𝑇 𝑅2
𝑉1 = 𝑉2 =
𝑅𝑇 𝑅𝑇

𝑅𝑇 = 𝑅1 + 𝑅2 𝑅𝑇 = 𝑅1 + 𝑅2
𝑉𝑇 𝑅1 𝑉𝑇 𝑅2
𝑉1 = 𝑉2 =
𝑅1 + 𝑅2 𝑅1 + 𝑅2
• The current flowing in each resistor is equal to the supply current
• The voltage is equal to the sum of the voltage drop across each resistance
• Series – equal current

Parallel Circuit

The total resistance is equal to the reciprocal of the sum of the reciprocals of
individual resistances.
Current Divider Equation
𝐼𝑇 = 𝐼1 = 𝐼2
𝑉𝑇 = 𝑉1 𝑉𝑇 = 𝑉1
𝑉𝑇 = 𝑉1 = 𝑉2
𝐼𝑇 𝑅𝑇 = 𝐼1 𝑅1 𝐼𝑇 𝑅𝑇 = 𝐼2 𝑅2
𝑉𝑇 𝑉1 𝑉2
= + 𝐼𝑇 𝑅𝑇 𝐼𝑇 𝑅𝑇
𝑅𝑇 𝑅1 𝑅2 𝐼1 = 𝐼2 =
𝑅1 𝑅2
𝑉𝑇 𝑉𝑇 𝑉𝑇 𝑅1 𝑅2 𝑅1 𝑅2
= + 𝑅𝑇 = 𝑅𝑇 =
𝑅𝑇 𝑅1 𝑅2 𝑅1 + 𝑅2 𝑅1 + 𝑅2
1 1 1 𝑅 𝑅2 𝑅1 𝑅2
𝑉𝑇 ( ) = 𝑉𝑇 ( + ) 𝐼𝑇 (𝑅 1+ 𝑅 ) 𝐼𝑇 (
𝑅1 + 𝑅2
)
𝑅𝑇 𝑅1 𝑅2 1 2
𝐼1 = 𝐼2 =
𝑅1 𝑅1
1 1 1
= + 𝐼𝑇 𝑅1 𝑅2 𝐼𝑇 𝑅1 𝑅2
𝑅𝑇 𝑅1 𝑅2
𝐼1 = 𝐼2 =
(𝑅2 + 𝑅1 )𝑅1 (𝑅2 + 𝑅1 )𝑅2
1
𝑅1 + 𝑅2 𝐼𝑇 𝑅2 𝐼𝑇 𝑅1
𝑅𝑇 = 𝐼1 = 𝐼2 =
𝑅1 𝑅2 𝑅1 + 𝑅2 𝑅1 + 𝑅2
𝑅1 𝑅2
𝑅𝑇 =
𝑅1 + 𝑅2
• The voltage drop across each resistor is equal to the total voltage
• The total current is equal to the sum of the currents flowing in each resistance.
• The equivalent resistance of two parallel resistors is equal to the product of their
resistances divided by their sum.
• The equivalent conductance of resistors connected in parallel is the sum of their
individual conductance.
Series Parallel
▪ Resistance = Ohm
▪ Conductance = Mho, Semens 𝐶1 𝐶2 𝐶𝑇 = 𝐶1 + 𝐶2
𝐶𝑇 =
1 𝐶1 + 𝐶2
𝐺= 𝐺𝑒𝑞 = 𝐺1 + 𝐺2 + ⋯ + 𝐺𝑁
𝑅
Sample Problem
1.)

𝑅1 = 1Ω ∫ 5Ω = 6Ω

𝑅2 = 6Ω‖3Ω
6×3
= = 2Ω
6+3

𝑅3 = 2Ω ∫ 2Ω = 4Ω

𝑅4 = 6Ω‖4Ω
6×4
= = 2.4Ω
6+4

𝑅𝑒𝑞 = 4Ω ∫ 2.4Ω ∫ 8Ω = 14.4Ω


2.)

𝑅1 = 4Ω ∫ 5Ω ∫ 3Ω = 12Ω

𝑅2 = 12Ω‖4Ω
12 × 4
= = 3Ω
12 + 4

𝑅3 = 3Ω ∫ 3Ω = 6Ω

𝑅4 = 6Ω‖6Ω
6×6
= = 3Ω
6+6

𝑅𝑒𝑞 = 2Ω ∫ 3Ω ∫ 1Ω = 6Ω
3.)

𝑅1 = 20Ω‖5Ω
20 × 5
= = 4Ω
20 + 5

𝑅2 = 4Ω ∫ 1Ω = 5Ω

𝑅3 = 20Ω‖5Ω
20 × 5
= = 4Ω
20 + 5

𝑅4 = 4Ω ∫ 2Ω = 6Ω

𝑅5 = 18Ω‖3.6Ω
18 × 3.6
= = 3Ω
18 + 3.6

𝑅5 = 6Ω‖9Ω
6×9
= = 3.6Ω
6+9

𝑅𝑒𝑞 = 8Ω ∫ 3Ω = 11Ω
Situations often arise in circuit analysis when the resistors are neither parallel nor in series.
These are the wye and delta network; these networks occur by themselves or as part of a
larger network.
Wye to Delta Connected Resistors
𝑅1 𝑅2 + 𝑅2 𝑅3 + 𝑅3 𝑅1
𝑅𝑎 =
𝑅1
𝑅1 𝑅2 + 𝑅2 𝑅3 + 𝑅3 𝑅1
𝑅𝑏 =
𝑅2
𝑅1 𝑅2 + 𝑅2 𝑅3 + 𝑅3 𝑅1
𝑅𝑐 =
𝑅3

• Each resistor in the delta network is the sum of all possible products of Y resistors
taken two at a time, divided by the opposite Y resistor.
Delta to Wye Connected Resistor
𝑅𝑏 𝑅𝑐
𝑅1 =
𝑅𝑎 + 𝑅𝑏 + 𝑅𝑐
𝑅𝑎 𝑅𝑐
𝑅2 =
𝑅𝑎 + 𝑅𝑏 + 𝑅𝑐
𝑅𝑎 𝑅𝑏
𝑅3 =
𝑅𝑎 + 𝑅𝑏 + 𝑅𝑐

Sample Problem
1.)

𝑅1 𝑅2 + 𝑅2 𝑅3 + 𝑅3 𝑅1 (10)(20) + (20)(40) + (40)(10)


𝑅𝑎 = = = 140Ω
𝑅1 10
𝑅1 𝑅2 + 𝑅2 𝑅3 + 𝑅3 𝑅1 (10)(20) + (20)(40) + (40)(10)
𝑅𝑏 = = = 70Ω
𝑅2 20
𝑅1 𝑅2 + 𝑅2 𝑅3 + 𝑅3 𝑅1 (10)(20) + (20)(40) + (40)(10)
𝑅𝑐 = = = 35Ω
𝑅3 40
2.) A circuit consisting of three resistors rated 10Ω, 15Ω, and 20Ω are connected in delta.
What would be the resistances of the equivalent wye connected load?

𝑅𝑏 𝑅𝑐 (15)(20)
𝑅1 = = = 6.67Ω
𝑅𝑎 + 𝑅𝑏 + 𝑅𝑐 (10) + (15) + (20)
𝑅𝑎 𝑅𝑐 (10)(20)
𝑅2 = = = 4.44Ω
𝑅𝑎 + 𝑅𝑏 + 𝑅𝑐 (10) + (15) + (20)
𝑅𝑎 𝑅𝑏 (10)(15)
𝑅3 = = = 3.33Ω
𝑅𝑎 + 𝑅𝑏 + 𝑅𝑐 (10) + (15) + (20)
3.)

𝑅𝑏 𝑅𝑐 (20)(30)
𝑅1 = = = 10Ω
𝑅𝑎 + 𝑅𝑏 + 𝑅𝑐 (10) + (20) + (30)

𝑅𝑎 𝑅𝑐 (10)(30)
𝑅2 = = = 5Ω
𝑅𝑎 + 𝑅𝑏 + 𝑅𝑐 (10) + (20) + (30)

𝑅𝑎 𝑅𝑏 (10)(20)
𝑅3 = = = 3.33Ω
𝑅𝑎 + 𝑅𝑏 + 𝑅𝑐 (10) + (20) + (30)

𝑅4 = 5Ω ∫ 3.33Ω = 8.33Ω
𝑅1 𝑅2 + 𝑅2 𝑅3 + 𝑅3 𝑅1 (10)(5) + (5)(8.33) + (8.33)(10)
𝑅𝑎 = = = 17.495Ω
𝑅1 10

𝑅1 𝑅2 + 𝑅2 𝑅3 + 𝑅3 𝑅1 (10)(5) + (5)(8.33) + (8.33)(10)


𝑅𝑏 = = = 34.99Ω
𝑅2 5

𝑅1 𝑅2 + 𝑅2 𝑅3 + 𝑅3 𝑅1 (10)(5) + (5)(8.33) + (8.33)(10)


𝑅𝑐 = = = 21.00Ω
𝑅3 8.33

𝑅4 = 12.5Ω‖17.495Ω
12.5 × 17.495
=
12.5 + 17.495
= 7.29Ω

𝑅5 = 15Ω‖34.99Ω
15 × 34.99
=
15 + 34.99
= 10.5Ω

𝑅6 = 7.29Ω ∫ 10.5Ω = 17.79Ω

𝑅𝑒𝑞 = 17.79Ω‖21Ω
17.79 × 21
= = 9.63Ω
17.79 + 21

𝑉𝑇
𝐼𝑇 =
𝑅𝑇
120𝑉
= = 12.46𝐴
9.63Ω
Kirchhoff’s Law

- first introduced in 1847 by the German physicist Gustav Robert


Kirchhoff (1824-1887). These laws are formerly known as Kirchhoff’s
Current Law (KCL) and Kirchhoff’s Voltage Law (KVL).

Kirchhoff’s Current Law (KCL) – states that the algebraic sum of currents entering and
leaving the anode (or a close boundary) is zero.

– the sum of the current entering anode is equal to the sum


of the currents leaving the node.

Kirchhoff’s Voltage Law (KVL) – states that the algebraic sum of all voltages around a
closed path (or loop) is zero.

– sum of voltage drops = sum of voltage rises

Branches, Nodes, and Loops

Branches – represents a single element such as voltage source, current source or a resistor.

– circuit elements
▪ Resistor
▪ Current Source

➢ Independent Current Source

➢ Dependent Current Source

▪ Voltage Source

➢ Independent Voltage Source – have definite value or


specific value

➢ Dependent Voltage Source – voltage value depends on the


other circuit elements

Branches

• 10V
• 5Ω
• 2Ω
• 3Ω
• 2A

Loops

• 12651
• 23762
• 34873
• 14851
Nodes – point of connection of two or more branches

▪ Non-essential – two elements


▪ Essential – more than three elements

General Node

• 1
• 2,3,4
• 5,6,7,8

Loops – any closed path within a circuit

KCL

Entering the node +

Leaving the node -

𝐼1 − 𝐼2 − 𝐼3 = 0

𝐼1 = 𝐼2 + 𝐼3

Entering Leaving

KVL

𝑉𝑇 − 𝑉1 − 𝑉2 = 0

𝑉𝑇 = 𝑉1 + 𝑉2

Rise Voltage Drop

Nodes with 3 or more resistor

1.) Solve for the currents and the voltages in the given circuit

KCL Equation = No. of Nodes – 1

=2-1=1

KCL at Node 1

𝐼1 − 𝐼2 − 𝐼3 = 0 - equation 1

By KVL at 𝐿1 V = IR

30 − 𝐼1 (8) − 𝐼2 (3) = 0

30 − 8𝐼1 − 3𝐼2 = 0

8𝐼1 + 3𝐼2 = 30 - equation 2


By KVL at 𝐿2 From equation 4
𝐼2 (3) + 𝐼3 (6) = 0 11𝐼2 + 8𝐼3 = 30
3𝐼2 + 6𝐼3 = 0 - equation 3 11(2𝐼3 ) + 8𝐼3 = 30
• 𝐼1 − 𝐼2 − 𝐼3 = 0 22𝐼3 + 8𝐼3 = 30
• 8𝐼1 + 3𝐼2 = 30
• 3𝐼2 + 6𝐼3 = 0 30𝐼3 = 30

Eliminate 𝐼1 30
𝐼3 = = 1𝐴
30
𝐼1 − 𝐼2 − 𝐼3 = 0
𝐼2 = 2𝐼3 = (2)(1) = 2𝐴
𝐼1 = 𝐼2 + 𝐼3 - substitute to equation 2
𝐼1 = 𝐼2 + 𝐼3 = 2 + 1 = 3𝐴
8𝐼1 + 3𝐼2 = 30
−𝑉1 + 𝐼1 (8) = 0
8(𝐼2 + 𝐼3 ) + 3𝐼2 = 30
𝑉1 = 8𝐼1 = 8(3) = 24𝑉
8𝐼2 + 8𝐼3 + 3𝐼2 = 30
−𝑉2 + 𝐼2 (3) = 0
11𝐼2 + 8𝐼3 = 30 - equation 4
𝑉2 = 3𝐼2 = 3(2) = 6𝑉
Eliminate 𝐼2 from equation 3
−𝑉3 + 𝐼3 (6) = 0
3𝐼2 + 6𝐼3 = 0
𝑉3 = 6𝐼1 = 6(1) = 6𝑉
6𝐼3
𝐼2 = = 2𝐼3 - substitute to equation 4
3

2.)

KCL at Node 2

𝐼1 + 𝐼3 − 𝐼2 − 𝐼4 = 0

𝐼1 − 𝐼2 + 𝐼3 − 𝐼4 = 0 – equation 1

KVL at 𝐿1

−𝐼1 (12) + 𝐼3 (6) = 0

−12𝐼1 + 6𝐼3 = 0

12𝐼1 − 6𝐼3 = 0 – equation 2

KVL at 𝐿2

15 − 𝐼3 (6) + 𝐼4 (10) = 0

15 − 6𝐼3 + 10𝐼4 = 0

6𝐼3 + 10𝐼4 = 15 – equation 3

KVL at 𝐿3

−𝐼2 (40) + 𝐼4 (10) = 0

−40𝐼2 + 10𝐼4 = 0

40𝐼2 − 10𝐼4 = 0 – equation 4


• 𝐼1 − 𝐼2 + 𝐼3 − 𝐼4 = 0 Find 𝐼2
• 12𝐼1 − 6𝐼3 = 0
• 6𝐼3 + 10𝐼4 = 15 𝐼2 = 0.25𝐼4
• 40𝐼2 − 10𝐼4 = 0
= 0.25(1) = 0.25𝐴
Eliminate 𝐼1 from equation 2
Find 𝐼1
12𝐼1 − 6𝐼3 = 0
𝐼1 = 0.5𝐼3
6𝐼3
𝐼1 = = 0.5𝐼3 - substitute to equation 1 5 5
12
= 0.5 ( ) = 𝐴
6 12
𝐼1 − 𝐼2 + 𝐼3 − 𝐼4 = 0
Find 𝑉1
(0.5𝐼3 ) − 𝐼2 + 𝐼3 − 𝐼4 = 0
𝑉1 − 𝐼1 (12) = 0
−𝐼2 + 1.5𝐼3 − 𝐼4 = 0 – equation 5
𝑉1 = 𝐼1 (12)
Eliminate 𝐼2 from equation 4
5
40𝐼2 − 10𝐼4 = 0 = (12) = 5𝑉
12
10𝐼4
𝐼2 = = 0.25𝐼4 - substitute to equation 5 Find 𝑉2
40

−𝐼2 + 1.5𝐼3 − 𝐼4 = 0 𝑉2 − 𝐼2 (40) = 0

−(0.25𝐼4 ) + 1.5𝐼3 − 𝐼4 = 0 𝑉2 = 𝐼2 (40)

1.5𝐼3 − 1.25𝐼4 = 0 – equation 6 = 0.25(40) = 10𝑉

Eliminate 𝐼3 from equation 3

6𝐼3 + 10𝐼4 = 15
15−10𝐼4
𝐼3 = - substitute to equation 6
6

1.5𝐼3 − 1.25𝐼4 = 0
15 − 10𝐼4
6 [1.5 ( ) − 1.25𝐼4 = 0] 6
6
1.5(15 − 10𝐼4 ) − 7.5𝐼4 = 0

22.5 − 22.5𝐼4 = 0
22.5
𝐼4 = = 1𝐴
22.5
Find 𝐼3
15 − 10𝐼4
𝐼3 =
6
15 − 10(1) 5
𝐼3 = = 𝐴
6 6
3.)

KVL at 𝐿1

24 − 𝑉1 + 𝑉2 = 0

𝑉1 − 𝑉2 = 24 – equation 1

KVL at 𝐿2

−12 − 𝑉2 − 𝑉3 = 0

𝑉2 + 𝑉3 = −12 – equation 2

KVL at 𝐿3

−10 + 𝑉3 = 0

𝑉3 = 10𝑉 - substitute to equation 2

𝑉2 + 𝑉3 = −12

𝑉2 + 10 = −12

𝑉2 = −12 − 10 = 22 𝑉- substitute to equation 1

𝑉1 − 𝑉2 = 24

𝑉1 = 24 + 𝑉2

= 24 + (−22)

= 24 − 22 = 2 𝑉

4.) Solve for 𝑉0

KCL at Node 1

−𝐼1 + 10 + 2𝑉0 = 0

𝑉0 + 𝐼1 (4) = 0

𝑉0 = −𝐼1 (4)

= −4𝐼1

−𝐼1 + 10 + 2(−4𝐼1 ) = 0

−𝐼1 + 10 − 8𝐼1 = 0

−9𝐼1 + 10 = 0

9𝐼1 = 10
10
𝐼1 = 𝐴
9
𝑉0 = −𝐼1 (4)
10 40
= −4 ( )=−
9 9
• With only voltage source connected to a node, KCL is not applicable
• With only current connected to a node, KVL is not applicable

Nodal Analysis – provides a general procedure for analyzing circuits using node voltages as
the circuit variables. It is also known as the node voltage method. In nodal analysis, we are
interested in finding the node voltages.

Steps to Determine Node Voltages:

1. Select a node as the reference node. Assign voltages 𝑉1 , 𝑉2 …, 𝑉𝑛 − 1 to the


remaining n-1 nodes. The voltages are referenced with respect to the reference
node.
2. Apply KCL to each of the n-1 non reference nodes. Use Ohm’s Law to express the
branch currents in terms of node voltages.
𝑖𝑛𝑎𝑙𝑖𝑠𝑎𝑛 − 𝑝𝑖𝑛𝑢𝑛𝑡𝑎ℎ𝑎𝑛
𝑑𝑖𝑛𝑎𝑎𝑛𝑎𝑛 (𝑅𝑒𝑠𝑖𝑠𝑡𝑎𝑛𝑐𝑒)
3. Solve the resulting simultaneous equations to obtain the unknown node voltages.

Sample Problems

1.)

KCL at Node 1 (𝑉1 )

−𝐼1 +𝐼2 + 5 = 0

𝐼1 −𝐼2 = 5
𝑉1 − 𝑉0 𝑉2 − 𝑉1
[( )−( ) = 5]4
2 4
5𝑉2 − 3𝑉1 = 60 Solving for the currents
2𝑉1 − 2𝑉0 − (𝑉2 − 𝑉1 ) = 20
20 + 𝑉2 𝑉1 − 𝑉0
5𝑉2 − 3( ) = 60 𝐼1 =
2𝑉1 − 𝑉2 + 𝑉1 = 20 3 2
3𝑉1 − 𝑉2 = 20 – equation 1 5𝑉2 − (20 + 𝑉2 ) = 60 40
−0
= 3
KCL at Node 2 (𝑉2 ) 5𝑉2 − 20 − 𝑉2 = 60 2

−𝐼2 −𝐼3 + 10 − 5 = 0 40
4𝑉2 − 20 = 60 𝐼1 = 𝐴
6
−𝐼2 −𝐼3 + 5 = 0 4𝑉2 = 60 + 20
𝑉2 − 𝑉1
𝐼2 =
𝐼2 +𝐼3 = 5 4𝑉2 80 4
=
4 4 40
𝑉2 − 𝑉1 𝑉2 − 𝑉0 20 − 3
[( )+( ) = 5]12
4 6 𝑉2 = 20𝑉 =
4
(3𝑉2 − 3𝑉1 ) + 2𝑉2 − 2𝑉0 = 60 20 + 𝑉2
𝑉1 = 5
3 𝐼2 = 𝐴
3𝑉2 − 3𝑉1 + 2𝑉2 = 60 3
20 + 20
= 𝑉2 − 𝑉0
5𝑉2 − 3𝑉1 = 60 – equation 2 3 𝐼3 =
6
Eliminate 𝑉1 from equation 1 40
𝑉1 = 𝑉 20 − 0
3 =
3𝑉1 − 𝑉2 = 20 6

20 + 𝑉2 20
𝑉1 = - substitute to equation 2 𝐼3 = 𝐴
3 6
2.)

KCL at Node 1

−𝐼1 −𝐼2 + 1 = 0

𝐼1 +𝐼2 = 1
𝑉1 − 𝑉0 𝑉1 − 𝑉2
[( )+( ) = 1]6
2 6
3𝑉1 − 3𝑉0 + (𝑉1 − 𝑉2 ) = 6

3𝑉1 + 𝑉1 − 𝑉2 = 6 Eliminate 𝑉1 from equation 1 −45𝑉2 = 672 − 42


4𝑉1 − 𝑉2 = 6 – equation 1 4𝑉1 − 𝑉2 = 6 −45𝑉2 630
=
6 + 𝑉2 −45 −45
KCL at Node 2
𝑉1 = - substitute to equation 2
4 𝑉2 = −14 𝑉
𝐼2 −𝐼3 − 4 = 0
7𝑉1 − 13𝑉2 = 168 6 + (−14)
𝐼2 −𝐼3 = 4 𝑉1 =
6 + 𝑉2 4
𝑉1 − 𝑉2 𝑉2 − 𝑉0 7( ) − 13𝑉2 = 168
[( )−( ) = 4]42 4 6 − 14
6 7 =
42 + 7𝑉2 4
(7𝑉1 − 7𝑉2 ) − (6𝑉2 − 6𝑉0 ) = 168 [ − 13𝑉2 = 168] 4
4 8
=−
4
7𝑉1 − 7𝑉2 − 6𝑉2 = 168 42 + 7𝑉2 − 52𝑉2 = 672
𝑉1 = −2 𝑉
7𝑉1 − 13𝑉2 = 168 – equation 2 42 − 45𝑉2 = 672

3.)

KCL at Node 1

3 − 𝐼𝑋 −𝐼2 = 0

𝐼𝑋 +𝐼2 = 3
𝑉1 − 𝑉2 𝑉1 − 𝑉3
[( )+( ) = 3]4
2 4
2𝑉1 − 2𝑉2 + (𝑉1 − 𝑉3 ) = 12

3𝑉1 − 2𝑉2 − 𝑉3 = 12 – equation 1

KCL at Node 2

𝐼𝑋 − 𝐼1 −𝐼3 = 0
𝑉1 − 𝑉2 𝑉2 − 𝑉0 𝑉2 − 𝑉3
[( )−( )−( ) = 0]8
2 4 8
4𝑉1 − 4𝑉2 − 2𝑉2 + 2𝑉0 − 𝑉2 + 𝑉3 = 0

4𝑉1 − 7𝑉2 + 𝑉3 = 0 – equation 2


KCL at Node 3 △ 𝑉1
𝑉1 =
𝐼2 +𝐼3 − 2𝐼𝑋 = 0 △
48
𝑉1 − 𝑉3 𝑉2 − 𝑉3 𝑉1 − 𝑉2 = = 4.8𝑉
[( )−( ) −2( ) = 0]8 10
4 8 2
2𝑉1 − 2𝑉3 + 𝑉2 − 𝑉3 − 8𝑉1 + 8𝑉2 = 0
△ 𝑉2
[−6𝑉1 + 9𝑉2 − 3𝑉3 = 0] ÷ 3 𝑉2 =

−2𝑉1 + 3𝑉2 − 𝑉3 = 0 – equation 3 24
= = 2.4𝑉
• 3𝑉1 − 2𝑉2 − 𝑉3 = 12 10
• 4𝑉1 − 7𝑉2 + 𝑉3 = 0
• −2𝑉1 + 3𝑉2 − 𝑉3 = 0
△ 𝑉3
Using Cramer’s Rule 𝑉3 =

−24
𝑉1 𝑉2 𝑉3 = = −2.4𝑉
10
3 −2 −1 𝑉1 12
4 −7 1 𝑉2 = 0
−2 3 −1 𝑉3 0
𝑉1 − 𝑉2
3 −2 −1 3 −2 𝐼𝑋 =
2
△= 4 −7 1 4 −7
−2 3 −1 −2 3 4.8 − 2.4
= = 1.2𝐴
2
△ = (21 + 4 − 12) − (−14 + 9 + 8)

= 13 − 3 = 10
𝑉2 − 𝑉0
12 −2 −1 12 −2 𝐼1 =
4
△ 𝑉1 = 0 −7 1 0 −7
0 3 −1 0 3 2.4 − 0
= = 0.6𝐴
4
△ 𝑉1 = (84 + 0 + 0) − (0 + 36 + 0)

= 84 − 36 = 48
𝑉1 − 𝑉3
3 12 −1 3 12 𝐼2 =
4
△ 𝑉2 = 4 0 1 4 0
−2 0 −1 −2 0 4.8 − (−2.4)
= = 1.8𝐴
4
△ 𝑉2 = (0 − 24 + 0) − (0 + 0 − 48)

= −24 + 48 = 24
𝑉2 − 𝑉3
3 −2 12 3 −2 𝐼3 =
8
△ 𝑉3 = 4 −7 0 4 −7
−2 3 0 −2 3 2.4 − (−2.4)
= = 0.6𝐴
△ 𝑉3 = (0 + 0 + 144) − (168 + 0 + 0) 8

= 144 − 168 = −24


4.)

KCL at Node 1

10 − 𝐼2 −𝐼2 = 0

𝐼1 +𝐼2 = 10
𝑉1 − 𝑉2 𝑉1 − 𝑉3
[( )+( ) = 10]6
3 2
2𝑉1 − 2𝑉2 + 3𝑉1 − 3𝑉3 = 60

5𝑉1 − 2𝑉2 − 3𝑉3 = 60 – equation 1

KCL at Node 2

𝐼1 − 𝐼𝑋 + 4𝐼𝑋 = 0

𝐼1 + 3𝐼𝑋 = 0
𝑉1 − 𝑉3 𝑉2 − 𝑉0
[( )+ 3( ) = 0]12
3 4
4𝑉1 − 4𝑉2 + 9𝑉2 − 9𝑉0 = 0

4𝑉1 + 5𝑉2 = 0 – equation 2

KCL at Node 3

𝐼2 − 4𝐼𝑋 − 𝐼3 = 0
𝑉1 − 𝑉3 𝑉2 − 𝑉0 𝑉3 − 𝑉0
[( )− 4( )−( ) = 0]12
2 4 6
6𝑉1 − 6𝑉3 − 12𝑉2 + 12𝑉0 − 2𝑉3 + 3𝑉0 = 0

[6𝑉1 − 12𝑉2 − 8𝑉3 = 0] ÷ 2

3𝑉1 − 6𝑉2 − 4𝑉3 = 0 – equation 3

Using Equation Mode in Calculator

𝑉1 = 80𝑉

𝑉2 = −64𝑉

𝑉3 = 156𝑉

Nodal Analysis with Source Voltages

Case 1: If a voltage source is connected between the reference node and a non-reference
node, we simply set the voltage at the non-reference node equal to the voltage of the voltage
source.

Case 2: If the voltage source (dependent or independent) is connected between two non-
reference nodes, the two non-reference nodes form a generalized node or supernode; we
apply both KCL and KVL to determine the node voltages.
Supernode

A supernode is formed by enclosing a (dependent or independent) voltage source connected


between two non-reference nodes and any elements connected in parallel with it.

Sample Problem

1.)

KCL at SN 𝑉1 and 𝑉2

2 − 𝐼1 − 𝐼2 −𝐼3 + 𝐼2 − 7 = 0

−𝐼1 − 𝐼3 − 5 = 0

𝐼1 + 𝐼3 = −5
𝑉1 − 𝑉0 𝑉2 − 𝑉0
[( )+( ) = −5]4
2 4
2𝑉1 − 2𝑉0 + 𝑉2 − 𝑉0 = −20

2𝑉1 + 𝑉2 = −20 – equation 1

KVL at 𝑆𝐿1

𝑉1 + 2 − 𝑉2 = 0

𝑉1 − 𝑉2 = −2 – equation 2

Solve for Node Voltages

𝑉2 = 𝑉1 + 2 = −20 − 2𝑉1

3𝑉1 = −22
22
𝑉1 = −
3
𝑉1 = −7.33 𝑉

𝑉2 = 𝑉1 + 2
22
=− +2
3
16
=−
3
𝑉2 = −5.33 𝑉
2.)

KCL at SN 𝑉1 and 𝑉2

−𝐼1 − 𝐼2 +10 − 𝐼3 = 0

𝐼1 + 𝐼2 + 𝐼3 = 10
𝑉1 − 𝑉0 𝑉1 − 𝑉4 𝑉2 − 𝑉3
[( )+( )+( ) = 10]6
2 3 6
3𝑉1 − 3𝑉0 + 2𝑉1 − 2𝑉4 + 𝑉2 − 𝑉3 = 60

5𝑉1 + 𝑉2 − 𝑉3 − 2𝑉4 = −20 – equation 1

KCL at SN 𝑉3 and 𝑉4

𝐼3 − 𝐼4 +𝐼2 − 𝐼5 = 0

𝐼2 + 𝐼3 −𝐼4 −𝐼5 = 0
𝑉1 − 𝑉4 𝑉2 − 𝑉3 𝑉3 − 𝑉0 𝑉4 − 𝑉0
[( )+( )−( )−( ) = 0]12
3 6 4 1
4𝑉1 − 4𝑉4 + 2𝑉2 − 2𝑉3 − 3𝑉3 + 3𝑉0 − 12𝑉4 + 12𝑉0 = 0

4𝑉1 + 2𝑉2 − 5𝑉3 − 16𝑉4 = 0 – equation 2 From equation 3


KVL at 𝐿1 𝑉1 − 𝑉2 = 20
𝑉1 − 20 − 𝑉2 = 0 𝑉2 = 𝑉1 − 20 – substitute to equation 1 & 2
𝑉1 − 𝑉2 = 20 – equation 3 5𝑉1 + (𝑉1 − 20) − 𝑉3 − 2𝑉4 = −20
6𝑉1 − 5𝑉3 − 16𝑉4 = 40 – equation 4 6𝑉1 − 𝑉3 − 2𝑉4 = 80 – equation 4
KVL at 𝐿2 4𝑉1 + 2(𝑉1 − 20) − 5𝑉3 − 16𝑉4 = 0
𝑉3 − 3𝑉𝑋 − 𝑉4 = 0 Using Equation Mode in Calculator
𝑉3 − 3(3𝐼2 ) − 𝑉4 = 0 80
𝑉1 = 𝑉 𝑜𝑟 26.6667𝑉
𝑉1 − 𝑉4 3
𝑉3 − 9 ( ) − 𝑉4 = 0
3 𝑉2 = 𝑉1 − 20
𝑉3 − 3𝑉1 + 3𝑉4 − 𝑉4 = 0 80 20
= − 20 = 𝑜𝑟 6.6667𝑉
[−3𝑉1 + 𝑉3 + 2𝑉4 = 0] − 1 3 3
520
3𝑉1 − 𝑉3 − 2𝑉4 = 0 – equation 3 𝑉3 = 𝑉 𝑜𝑟 173.3333𝑉
3
140
𝑉4 = − 𝑉 𝑜𝑟 46.6667𝑉
3

Mesh Analysis – provides another general procedure for analyzing circuits, using mesh
currents as the circuit variables. A mesh is a loop that does not contain any other loop within
it. Nodal analysis applies KCL to find unknown voltages in a given circuit, while mesh
analysis applies KVL to find unknown currents.
Steps to Determine Mesh Currents:

1.) Assign mesh currents 𝑖1 , 𝑖2 ,…, into the n meshes.


2.) Apply KVL to each of the n meshes. Use Ohm’s Law to express the voltages in
terms of the mesh currents.
3.) Solve the resulting n simultaneous equations to get the mesh currents.

Sample Problems

1.)

KVL at Mesh 𝑖1

15 − 𝑖1 (5 + 10) − 10 + 𝑖2 (10) = 0

5 − 15𝑖1 + 10𝑖2 = 0

[15𝑖1 − 10𝑖2 = 5] ÷ 5

3𝑖1 − 2𝑖2 = 1 – equation 1

KVL at Mesh 𝑖2 3(2𝑖2 − 1 ) − 2𝑖2 = 1 𝐼1 = 𝑖1 = 1𝐴


10 − 𝑖2 (10 + 6 + 4) + 𝑖1 (10) = 0 6𝑖2 − 3 − 2𝑖2 = 1 𝐼2 = 𝑖2 = 1𝐴
10 − 20𝑖2 + 10𝑖1 = 0 4𝑖2 = 4 𝐼1 − 𝐼2 − 𝐼3 = 0
[10𝑖1 − 20𝑖2 = −10] ÷ 10 𝑖2 = 1𝐴 𝐼3 = 𝐼1 − 𝐼2
𝑖1 − 2𝑖2 = −1 – equation 2 𝑖1 = 2𝑖2 − 1 =1−1
Eliminate 𝑉1 from equation 2 = 2(1) − 1 𝐼3 = 0𝐴
𝑖1 − 2𝑖2 = −1 =2−1
𝑖1 = 2𝑖2 − 1 - substitute to equation 1 𝑖1 = 1𝐴

2.)

KVL at Mesh 𝑖1

24 − 𝑖1 (10 + 12) + 𝑖2 (10) + 𝑖3 (12) = 0

24 − 22𝑖1 + 10𝑖2 + 12𝑖3 = 0

[22𝑖1 − 10𝑖2 + 12𝑖3 = 24] ÷ 2

11𝑖1 − 5𝑖2 + 6𝑖3 = 12 – equation 1

KVL at Mesh 𝑖2

−𝑖2 (4 + 10 + 24) + 𝑖3 (4) + 𝑖1 (10) = 0

−38𝑖2 + 4𝑖3 + 10𝑖1 = 0

[10𝑖1 − 38𝑖2 + 4𝑖3 = 0] ÷ 2

5𝑖1 − 19𝑖2 + 2𝑖3 = 0 – equation 2


KVL at Mesh 𝑖3

−4𝑖0 −𝑖3 (12 + 4) + 𝑖1 (12) + 𝑖2 (4) = 0

−4𝑖0 −16𝑖3 + 12𝑖1 + 4𝑖2 = 0

12𝑖1 + 4𝑖2 −16𝑖3 − 4𝑖0 = 0

KCL at Node A

𝐼1 − 𝐼2 − 𝐼0

𝐼0 = 𝐼1 − 𝐼2

12𝐼1 + 4𝐼2 −16𝐼3 − 4(𝐼1 − 𝐼2 ) = 0

12𝐼1 + 4𝐼2 −16𝐼3 − 4𝐼1 + 4𝐼2 = 0

[8𝐼1 + 8𝐼2 −16𝐼3 = 0] ÷ 8

𝐼1 + 𝐼2 − 2𝐼3 = 0 – equation 3

Using Equation Mode in Calculator


9
𝑖1 = 𝐴
4
3
𝑖2 = 𝐴
4
3
𝑖3 = 𝐴
2
𝑖0 = 𝑖1 − 𝑖2
9 3 3
𝑖0 = − =
4 4 2
3
4𝑖0 = 4( )
2
4𝑖0 = 6𝐴

Mesh Analysis with Current Sources

Case 1: When a current source exists only in one mesh.


Case 2: When a current source exists between two meshes.

b.)

a.)
Sample Problem

KVL at SM 𝑖1 and 𝑖2

6 − 𝑖1 (2) − 𝑖2 (4 + 8) + 𝑖3 (4) + 𝑖3 (2) = 0

6 − 2𝑖1 − 12𝑖2 + 6𝑖3 = 0

[−2𝑖1 − 12𝑖2 + 6𝑖3 = −6] ÷ −2

𝑖1 + 6𝑖2 − 3𝑖3 = 3 – equation 1

KVL at Mesh 𝑖3

−𝑖3 (2 + 2 + 4) + 𝑖1 (2) + 𝑖2 (4) = 0

−8𝑖3 + 2𝑖1 + 4𝑖2 = 0

[2𝑖1 + 4𝑖2 − 8𝑖3 = 0] ÷ 2

𝑖1 + 2𝑖2 − 4𝑖3 = 0 – equation 2

KCL at Node B

3 + 𝐼2 − 𝐼1 = 0

−𝐼1 + 𝐼2 = −3

𝐼1 − 𝐼2 = 3 – equation 3

Using Equation Mode in Calculator

𝑖1 = 3.4737𝐴

𝑖2 = 0.4737𝐴

𝑖3 = 1.1053𝐴

Superposition principle states that the voltage across (or current through) an element in a
linear circuit is the algebraic sum of the voltages across (or currents through) that element
due to each independent source acting alone.
Steps to Apply Superposition Principle

1.) Turn off all independent sources except one source. Find the output (voltage or
current) due to that active source using nodal or mesh analysis.
2.) Repeat step 1 for each of the other independent sources
3.) Find the total contribution by adding algebraically all the contributions due to the
independent sources.

Voltage Source: shorted

Current Source: open

Sample Problem

1.)

Let 6V acts alone; 3A turn off

6𝑉 − 𝑖1 (8 + 4) = 0 −𝑉1 + 𝑖1 (4)
12𝑖1 = 6 − 12𝑖2 + 6𝑖3 = 0 𝑉1 = 0.5(4) = 2𝑉
6
𝑖1 = = 0.5𝐴
12

Let 3A acts alone; 6V turn off

3(8) 𝑉 = 𝑉1 + 𝑉2
𝑖2 = = 2𝐴
4+8
= 2𝑉 + 8𝑉
𝑉2 = 2𝐴(4) = 8𝑉
𝑉 = 10𝑉
2.)

Let 8A acts alone; 20V turn off

KCL at Node 𝑉1 𝑉1 − 𝑉0
𝐼1 =
−𝐼1 + 𝐼2 + 8 = 0 5
20 − 0
𝐼1 + 𝐼2 = 8 =
5
𝑉1 − 𝑉0 𝑉1 − 𝑉0
( )+( )=8 𝐼1 = 4𝐴
5 5
2𝑉1
=8
5
𝑉1 = 20𝑉

Let 20V acts alone; 8A turn off

20(2) 𝑉0 = 𝑉0 1 + 𝑉0 2
𝑉0 1 = = 4𝑉
2+3+5
𝑉0 2 = 𝐼1 (2)
= 4𝑉 + 8𝑉
𝑉0 = 12𝑉
= 4(2) = 8𝑉
Source Transformation is the process of replacing a voltage source Vs in series with a
resistor R by a current source is in parallel with a resistor R or vice versa. It is also applicable
to dependent sources provided we carefully handle the dependent variable.

Sample Problem

1.)

Using Ohm’s Law


𝑉 = 𝐼𝑅

= 3𝐴(4Ω)

𝑉 = 12𝑉

𝑅 = 4Ω ∫ 2Ω = 6Ω

Using Ohm’s Law


𝑉
𝐼=
𝑅
12𝑉
𝐼1 = = 2𝐴
6Ω
12𝑉
𝐼2 = = 4𝐴
3Ω

𝐼𝑇 = 4𝐴 − 2𝐴 = 2𝐴

Using Current Divider


2(2)
𝐼1 = = 0.4𝐴
2+8
𝑉0 = 𝐼1 (8)

= 0.4(8)

𝑉0 = 3.2𝑉
2.)

𝑅 = 6Ω‖3Ω
6×3
= = 2Ω
6+3

Using Ohm’s Law


𝑉 = 𝐼𝑅

= 5𝐴(2Ω)

𝑉 = 10𝑉

𝑉𝑇 = 10𝑉 + 5𝑉 = 15𝑉

Using Ohm’s Law


𝑉
𝐼=
𝑅
15𝑉
𝐼1 = = 7.5𝐴
2Ω

𝑅 = 1Ω ∫ 4Ω = 5Ω

𝐼𝑇 = 7.5𝐴 + 3𝐴 = 10.5𝐴

𝑅 = 5Ω‖2Ω
5 × 2 10
= = = 1.43Ω
5+2 7
Using Current Divider
10
10.5( 7 )
𝑖0 =
10
7 +7
𝑖0 = 1.7797𝐴
Thevenin’s Theorem states that a linear two-terminal circuit can be replaces by an equivalent
circuit consisting of a voltage source 𝑉𝑇𝐻 in series with a resistor 𝑅𝑇𝐻 , where 𝑉𝑇𝐻 is the open-
circuit voltage at the terminals and 𝑅𝑇𝐻 is the input or equivalent resistance at the terminals
when the independent sources are turned off.
Steps in Finding 𝑹𝑻𝑯of a given circuit
Case 1: If the network has no dependent sources, we turn off all independent sources. 𝑅𝑇𝐻
is the input resistance of the network looking between terminals a and b.
Solving for the Load Current:
𝑉𝑇𝐻
𝐼𝐿 =
𝑅𝑇𝐻 + 𝑅𝐿
𝑅𝐿
𝑉𝐿 = 𝑅𝐿 𝐼𝐿 = 𝑉
𝑅𝑇𝐻 + 𝑅𝐿 𝑇𝐻

Sample Problem

1.) Find the Thevenin equivalent circuit of the circuit in the figure below, to the left of the
terminals a-b. Then find the current through RL = 6Ω.

𝑖1

4(12)
𝑅𝑇𝐻 = +1
4 + 12
𝑅𝑇𝐻 = 4Ω

KVL at Mesh 𝑖1

32−𝑖1 (4 + 12) + 𝑖2 (12) = 0

32 − 16𝑖1 + 12𝑖2 = 0

[16𝑖1 − 12𝑖2 = 32] ÷ 4

4𝑖1 − 3𝑖2 = 8 – equation 1

𝑖2 = −2𝐴 – substitute to equation 1


4𝑖1 − 3(−2) = 8 𝑉𝑇𝐻
𝐼𝐿 =
𝑅𝑇𝐻 + 𝑅𝐿
4𝑖1 + 6 = 8
30𝑉
4𝑖1 = 8 − 6 =
4Ω + 6Ω
[4𝑖1 = 2] ÷ 4 𝐼𝐿 = 3𝐴
2
𝑖1 = = 0.5𝐴
4

KCL at Node A

𝐼1 − 𝑖2 − 𝑖3 = 0

𝐼3 = 𝑖1 − 𝑖2
𝐼3 = 0.5 − (−2)
𝐼3 = 2.5𝐴
𝑉𝑇𝐻 = 𝐼3 (12)

= 2.5(12)

𝑉𝑇𝐻 = 30𝑉

2.) Using Thevenin’s theorem, find the equivalent circuit to the left of the terminals in the
circuit in the given figure below. Then find i.

𝑖1 𝑖2

𝑅𝑇𝐻 = 6Ω + 6Ω = 12Ω
4Ω(12Ω)
𝑅𝑇𝐻 =
4Ω + 12Ω
𝑅𝑇𝐻 = 3Ω

KVL at SM (𝑖1 & 𝑖2 )

12−𝑖1 (6) − 𝑖2 (6 + 4) = 0

12 − 6𝑖1 − 10𝑖2 = 0

[6𝑖1 + 10𝑖2 = 12] ÷ 2

3𝑖1 + 5𝑖2 = 6 – equation 1


KCL at Node A

𝐼1 − 𝐼2 + 2 = 0

𝐼1 − 𝐼2 = −2 – equation 2
𝐼1 = 𝐼2 − 2 – substitute to equation 1
3(𝐼2 − 2 ) + 5𝑖2 = 6
3𝐼2 − 6 + 5𝐼2 = 6
𝑉𝑇𝐻
8𝐼2 = 6 + 6 𝑖=
𝑅𝑇𝐻 + 𝑅𝐿
12 6𝑉
𝐼2 = 𝐴 = 1.5𝐴 =
8 3Ω + 1Ω
𝑉𝑇𝐻 = 𝐼2 (4) 3
𝑖= 𝐴 = 1.5𝐴
= 1.5(4) 2

𝑉𝑇𝐻 = 6𝑉

Case 2: If the network has dependent sources, we turn off all independent sources. As with
superposition, dependent sources are not to be turned off because they are controlled by
circuit variables. We apply voltage source 𝑉0 at terminals a and b and determine the resulting
𝑉
current 𝐼0 . Then 𝑅𝑇𝐻 = 𝐼0. Alternatively, we may insert a current source 𝐼0 at terminals a-b
0
𝑉0
and find the terminal voltage 𝑉0 . Again 𝑅𝑇𝐻 = . Either of the two approaches will give the
𝐼0
same result. In either approach, we may assume any value of 𝑉0 and 𝐼0 . For example, we
may use 𝑉0 = 1𝑉 or 𝐼0 = 1𝐴, or even use unspecified values of 𝑉0 or 𝐼0 .

Sample Problem

1.) 𝑉0
𝑅𝑇𝐻 =
𝐼0

𝑉0 = 1𝑉

−𝑖1 (4) − 𝑉𝑋 = 0

𝑉𝑋 = −4𝑖1

KVL at Mesh 𝑖1 KVL at Mesh 𝑖2 KVL at Mesh 𝑖3

−𝑖1 (4 + 2 + 6) + 𝑖2 (2) + 𝑖3 (6) = 0 2𝑉𝑋 −𝑖2 (2) + 𝑖1 (2) = 0 −𝑖3 (6 + 2) − 1 + 𝑖1 (6) = 0

[−12𝑖1 + 2𝑖2 + 6𝑖3 = 0] ÷ −2 2𝑉𝑋 −2𝑖2 + 2𝑖1 = 0 −8𝑖3 + 6𝑖1 = 1

6𝑖1 − 𝑖2 − 3𝑖3 = 0 – equation 1 2(−4𝑖1 )−2𝑖2 + 2𝑖1 = 0 6𝑖1 − 3𝑖3 = 1 – equation 3


−8𝑖1 −2𝑖2 + 2𝑖1 = 0

[−6𝑖1 −2𝑖2 = 0] ÷ −2

3𝑖1 +𝑖2 = 0 – equation 2


Using Equation Mode in Calculator KVL at Mesh 𝑖2
𝑖1 = −0.0556𝐴 −𝑖3 (2 + 6 + 4)+𝑖1 (4) + 𝑖2 (2)+= 0
𝑖2 = 0.1667𝐴 −12𝑖3 + 4𝑖1 + 2𝑖2 = 0
𝑖3 = −0.1667𝐴 [4𝑖1 + 2𝑖2 − 12𝑖3 = 0] ÷ 2
𝑖3 = −𝑖0 2𝑖1 + 𝑖2 − 6𝑖3 = 0
𝑖0 = −𝑖3 2(5) + 𝑖2 − 6𝑖3 = 0
𝑖0 = −(−0.1667) 10+𝑖2 − 6𝑖3 = 0
𝑖0 = 0.1667𝐴 𝑖2 − 6𝑖3 = −10 – equation 2
𝑉0 Using Equation Mode in Calculator
𝑅𝑇𝐻 =
𝐼0
𝑖2 = 10𝐴
1
= 10
1 𝑖3 = 𝐴
6 3

𝑅𝑇𝐻 = 6Ω 10
𝑉𝑇𝐻 = 𝐴(6Ω)
3
𝑖1 = 5𝐴
𝑉𝑇𝐻 = 20𝑉
KVL at Mesh 𝑖2

2𝑉𝑋 −𝑖2 (2) + 𝑖3 (2) = 0

2(4𝑖1 − 4𝑖3 )−2𝑖2 + 2𝑖3 = 0

8𝑖1 −8𝑖3 −2𝑖2 + 2𝑖3 = 0

[8𝑖1 −2𝑖2 −6𝑖3 = 0] ÷ 2

4𝑖1 −𝑖2 −3𝑖3 = 0

4(5)−𝑖2 −3𝑖3 = 0

20−𝑖2 −3𝑖3 = 0

𝑖2 +3𝑖3 = 20 – equation 1

2.)

𝐼0 = 1𝐴
𝑉0
𝑅𝑇𝐻 =
𝐼0
KCL at Node A

−2𝐼𝑋 − 𝐼1 + 𝐼𝑋 + 1 = 0 −𝐼𝑋 (2) − 𝑉0 = 0


𝐼𝑋 − 𝐼1 + 1 = 0 𝑉0 = −2𝐼𝑋
𝐼𝑋 + 𝐼1 = 1 = −2(2)
𝑉0 − 𝑉1 𝑉1 − 𝑉0 𝑉0 = −4𝑉
( )+( )=1
2 4
𝑉0
−𝑉1 𝑉1 𝑅𝑇𝐻 =
[ + = 1]4 𝐼0
2 4
−4𝑉
−2𝑉1 + 𝑉1 = 4 =
1𝐴
−𝑉1 = 4 𝑅𝑇𝐻 = −4Ω
𝑉1 = −4𝑉 𝑉𝑇𝐻 = −2𝐼𝑋
𝑉0 − 𝑉1
𝐼𝑋 =
2
0 − (−4)
=
2
𝐼𝑋 = 2𝐴

𝐼0 = 1𝐴

Norton’s Theorem states that a linear two-terminal circuit can be replaced by an equivalent
circuit consisting of a current source 𝐼𝑁 in parallel with a resistor 𝑅𝑁 , where 𝐼𝑁 is the short-
circuit current through the terminals and 𝑅𝑁 is the input or equivalent resistance at the
terminals when the independent sources are turned off.

For Norton’s Equivalent Resistance

𝑅𝑁 = 𝑅𝑇𝐻

For Norton’s Equivalent Current


𝑉
𝐼𝑁 = 𝑖𝑆𝐶 𝐼𝑁 = 𝑅𝑇𝐻
𝑇𝐻

Norton’s Equivalent Circuit


Sample Problem

KVL at SM (𝑖1 & 𝑖2 ) 𝑅𝑁 = 3Ω + 3Ω = 6Ω


15−𝑖1 (3) − 𝑖2 (3 + 6) = 0 6Ω(6Ω)
𝑅𝑁 =
15 − 3𝑖1 − 9𝑖2 = 0 6Ω + 6Ω
𝑅𝑁 = 3Ω
[3𝑖1 + 9𝑖2 = 15] ÷ 3

𝑖1 + 3𝑖2 = 5 – equation 1

KCL at Node A

𝐼1 − 𝐼2 + 4 = 0

𝐼1 − 𝐼2 = −4– equation 2

Using Equation Mode in Calculator


7
𝑖1 = − 𝐴 𝑜𝑟 − 1.75𝐴
4
9
𝑉𝑇𝐻 𝑉𝑇𝐻 2
𝑖2 = 𝐴 𝑜𝑟 2.25𝐴 𝑃𝑚𝑎𝑥 =
4 𝐼𝑁 = 4𝑅𝑇𝐻
𝑅𝑇𝐻
𝑉𝑇𝐻 = 𝑖2 (6) (13.5)2
13.5𝑉 =
𝐼𝑁 = 4(3)
= 2.25(6) 3Ω

𝑉𝑇𝐻 = 13.5𝑉 𝐼𝑁 = 4.5𝐴 𝑃𝑚𝑎𝑥 = 15.1875 𝑊

Maximum Power is transferred to the load resistance equals the Thevenin resistance as
seen from the load (𝑅𝐿 = 𝑅𝑇𝐻 ).

𝑉𝑇𝐻 2
𝑃𝑚𝑎𝑥 =
4𝑅𝑇𝐻

Sample Problem

Find the value of 𝑅𝐿 for maximum power transfer in the given circuit. Find the maximum
power.
6Ω(12Ω) KCL at Node A
𝑅𝑇𝐻 = + 3Ω + 2Ω
6Ω + 12Ω
𝐼1 − 𝐼2 − 𝐼3 = 0
𝑅𝑇𝐻 = 9Ω
𝐼3 = 𝐼1 − 𝐼2
KVL at Mesh 𝑖1
2
12−𝑖1 (6 + 12) + 𝑖2 (12) = 0 = − − (−2)
3
12 − 18𝑖1 + 12𝑖2 = 0 4
𝐼3 = 𝐴 𝑜𝑟 1.3333𝐴
[18𝑖1 − 12𝑖2 = 12] ÷ 6 3
𝑉𝑇𝐻 = 𝐼3 (12)
3𝑖1 − 2𝑖2 = 2 – equation 1
4
𝑖2 = −2𝐴 – substitute to equation 1 = (12)
3
3𝑖1 − 2(−2) = 2 𝑉𝑇𝐻 = 16𝑉
3𝑖1 + 4 = 2 𝑉𝑇𝐻 2
𝑃𝑚𝑎𝑥 =
3𝑖1 = −2 4𝑅𝑇𝐻

2 (16)2
𝑖1 = − 𝐴 =
3 4(9)

𝑃𝑚𝑎𝑥 = 7.1111 𝑊

Concepts and Problems

• According to Kirchhoff’s Current Law, what is the algebraic sum of all currents
entering and exiting a node?
o ZERO
• When the current flows through a heater coil, it glows but supply wiring does not
glow because…
o RESITANCE OF HEATER COIL IS MORE THAN THAT OF SUPPLY
WIRE
• Refers to the most important components in controlling flow of electrons
o VOLTAGE, RESISTANCE, CURRENT

• Which of the following is not a valid expression of Ohm's Law?


o V=IR
o R=PI
o R=V/I
o I=V/R
• The total resistance of two similar wire conductors connected in parallel is
o ONE HALF THE RESISTANCE OF ONE WIRE

• Which of the following allows more current if applied to the same voltage?
1
o 0.002 siemen = 0.002 = 500Ω
o 25 ohms = 25Ω
1
o 0.004 siemen = = 250Ω
0.004
o 2.5 ohms = 2.5Ω
• When you increase the resistance in a circuit, the flow of electrons will…
o BE DECREASED
• Ohm’s Law refers to…
o CURRENT VARIES DIRECTLY AS THE VOLTAGE AND INVERSELY AS
THE RESISTANCE
• A series circuit in which desired portions of the source voltage may be tapped off for
use equipment.
o VOLTAGE DIVIDER
• Other factors remaining constant, what would be the effect on the current flow in a
given circuit if the applied potential were doubled
o IT WOULD DOUBLE
• The hot resistance of an incandescent lamp is 10Ω and the rated voltage is 50V.
Find the series resistance required to operate the lamp from an 80V supply.
𝑉𝑆 = 𝑉1 + 𝑉2
80𝑉 = 50𝑉 + 𝑉2
𝑉2 = 80𝑉 − 50𝑉 = 30𝑉
𝑉 50𝑉
𝐼= = = 5𝐴
𝑅 10Ω
𝑉 30𝑉
𝑅= = = 6Ω
𝐼 5𝐴
• Two (2) 115-V incandescent lamps A and B are connected in series across a 230-
V source. If lamp A is rated 75 watts and lamp B is rated 50 watts, determine the
current drawn by the series connection. 𝑉𝑇 = 𝐼𝑇 𝑅𝑇
𝑉2 𝑉 2 1152 𝑅𝑇 = 𝑅1 + 𝑅2
𝑃= 𝑅1 = = = 176.33Ω 𝑉𝑇 230𝑉
𝑅 𝑃 75 𝐼𝑇 = =
𝑉2 2 2 𝑅 𝑇 = 176.33Ω + 264.5Ω 𝑅 440.83Ω
𝑉 115 𝑇
𝑅= 𝑅2 = = = 264.5Ω
𝑃 𝑃 50 𝑅𝑇 = 440.83Ω 𝐼𝑇 = 0.52𝐴
• A 5Ω resistance is connected in parallel with a 10Ω resistance. Another set, a 6Ω
and 8Ω resistance are connected in parallel. The two sets are connected in series.
What is the equivalent resistance?
5(10) 6(8)
𝑅𝑇 = + = 6.76Ω
5 + 10 6 + 8
• A 10Ω and a 20Ω resistance are connected in parallel. Another resistance of 5Ω is
connected in series with the two. If the supply voltage is 48V, what is the current
through the 10Ω resistor?
10(20)
𝑅𝑇 = + 5 = 11.67Ω
10 + 20
𝑉 48𝑉
𝐼𝑇 = = = 4.114𝐴
𝑅 11.67Ω
𝑉10&20 = 𝐼𝑅 = (4.114𝐴)(6.667Ω) = 27.43𝑉
𝑉10&20 27.43𝑉
𝐼10 = = = 2.743𝐴
𝑅10 10Ω
• Two resistances of 10 and 15Ω, each respectively, are connected in parallel. The
two are then connected in series with a 5Ω resistance. It is then connected across
a 12-V battery, what are the current and power?
15(10) 𝑃 = 𝐼𝑉
𝑅𝑇 = = 11Ω
15 + 10
= (1.09𝐴)(12𝑉)
𝑉𝑇 12𝑉
𝐼𝑇 = = = 1.09𝐴
𝑅𝑇 11Ω 𝑃 = 13.1𝑊
• Three resistors 10Ω, 15Ω, and 20Ω are connected in series across a 48V source.
What is the voltage across the 15Ω resistor?
𝑅𝑇 = 10Ω + 15Ω + 20Ω = 45Ω 𝑉15 = 𝐼𝑇 𝑅15
𝑉𝑇 48𝑉
𝐼𝑇 = = = 1.067𝐴 = (1.067𝐴)(15Ω) = 16𝑉
𝑅𝑇 45Ω
• The equivalent wye element of 3 equal resistors each equal to R and connected in
delta is:
𝑹
𝟑
• Referring to the circuit diagram below, if the charger voltage is 130V and the battery
voltage is 120V, solve for the current Ib.

KVL at Mesh 𝐿1
130−𝑖1 (3) − 𝑖2 (40) = 0
3𝑖1 − 40𝑖2 = 130 – equation 1
KVL at Mesh 𝐿2
40𝑖2 + 2𝑖𝑏 − 120 = 0
40𝑖2 + 2𝑖𝑏 = 120 – equation 2
KCL at Node 1
𝐼1 − 𝐼2 + 𝐼𝑏 = 0 – equation 3
𝐼1 = 3.20𝐴
𝐼2 = 3.009𝐴
𝐼𝑏 = −0.1942𝐴
• What is the equivalent resistance in ohms between points A and B of Fig. 16.7? All
resistance is in ohms.

12(6)
𝑅1 = = 4Ω
12 + 6
4(4)
𝑅𝑒𝑞 = = 2Ω
4+4
• A conductor has a resistance of 7Ω at 0C. At 20°C, the resistance has become 7.5Ω.
Calculate the temperature coefficient of the conductor at 20°C.
𝑅1 = 7Ω
𝑅2 = 7.5Ω
𝑡1 = 0°C
𝑡2 = 20°C 1
𝑎𝑡20 =
𝑅2 = 𝑅1 (1 + 𝛼𝑡20 △ 𝑇) 𝑇 + 𝑡20
1
7.5Ω = 7Ω[1 + 𝛼𝑡20 (20°C − 0°C)] =
280 + 20
7.5Ω = 7 + 140𝛼𝑡20
1
𝑎𝑡20 = /°𝐶
𝛼𝑡1 = 3.5714 × 10−3 /°𝐶 300
1
𝛼𝑡1 =
280

• The current through a branch in a linear network is 2A when the input source
voltage is 10 V. If the voltage is reduced to 1 V and the polarity is reversed, the
current through the branch is:
o -2A
o -0.2 A
o 0.2 A
o 2A
o 20 A
• The superposition theorem requires as many circuits to be solved as there are…
o SOURCES
• To determine the polarity of the voltage drop across a resistor, it is necessary to
know…
o DIRECTION OF CURRENT THROUGH THE RESISTOR
• The superposition theorem is applicable to
o BOTH CURRENT AND VOLTAGE
• For high efficiency of transfer of power, internal resistance of the source should be…
o EQUAL TO THE LOAD RESISTANCE
• For superposition, it is not required that only one independent source be considered
at a time; any number of independent sources may be considered simultaneously.
o FALSE
• The superposition principle applies to power calculation
o FALSE
• Refer to fig. 4.67. The Thevenin resistance at terminals a and b is:
o 5Ω
• In Thevenin’s Theorem to find Z
o ALL INDEPENDENT VOLTAGE SOURCES ARE SHORT CIRCUITED
AND ALL INDEPENDENT CURRENT SOURCES ARE OPEN CIRCUITED
• The Thevenin voltage across terminals a and b of the circuit in Fig. 4.67 is:
o 40V
• The Norton current at terminals a and b of the circuit in Fig. 4.67 is:
𝑉𝑇𝐻 50
𝐼𝑁 = = = 10𝐴
𝑅𝑇𝐻 5

o 10A
• The Norton resistance 𝑅𝑁 is exactly equal to the Thevenin resistance 𝑅𝑇𝐻
o TRUE
• Which pair of circuits in Fig. 4.69 are equivalent?
o a and c

• A load is connected to a network. At the terminals to which the load is connected,


𝑅𝑇𝐻 = 10Ω and 𝑉𝑇𝐻 = 40𝑉. The maximum possible power supplied to the load is:

𝑉𝑇𝐻 2 402
𝑃𝑚𝑎𝑥 = = = 40𝑊
4𝑅𝑇𝐻 4(10)

o 40W
• The source is supplying the maximum power to the load when the load resistance
equals the source resistance
o TRUE

• Find the branch currents of the given circuit. Use Elimination or Cramer’s Rule

KCL at Node 1

𝐼1 − 𝐼2 − 2 = 0

𝐼1 + 𝐼2 = 2 – equation 1
KCL at Node 2

2 − 𝐼3 − 𝐼4 − 𝐼5 = 0

𝐼3 + 𝐼4 − 𝐼5 = 2 – equation 2
KVL at 𝐿1 Eliminate 𝐼4 from equation 5

[30𝐼1 − 60𝐼2 = 0] ÷ 30 𝐼4 − 4𝐼5 = 8

𝐼1 − 2𝐼2 = 0 – equation 3 𝐼4 = 8 − 4𝐼5 - substitute to equation 6

KVL at 𝐿2 1.5(8 − 4𝐼5 ) + 𝐼5 = 2

[50𝐼3 − 25𝐼4 = 0] ÷ 25 12 − 6𝐼5 − 𝐼5

2𝐼3 − 𝐼4 = 0 – equation 4 7𝐼5 = 12

KVL at 𝐿3 12
𝐼5 = 𝐴
7
25𝐼4 − 100𝐼5 − 200 = 0
12
[25𝐼4 − 100𝐼5 = 200] ÷ 25 1.5𝐼4 − =2
7
𝐼4 − 4𝐼5 = 8 – equation 5 12
𝐼4 = 7
Eliminate 𝐼1 from equation 1 1.5
𝐼1 + 𝐼2 = 2 8 4
𝐼4 = 𝐴 𝐼1 = − 𝐴
7 3
𝐼1 = −2 − 𝐼2 - substitute to equation 3
𝐼4 2
𝐼3 = 𝐼2 = − 𝐴
(−2 − 𝐼2 ) − 2𝐼2 = 0 2 3

−2 − 3𝐼2 = 0 8 4
𝐼3 = 𝐴
𝐼3 = 7 7
2 2
𝐼2 = − 𝐴 8
3 4 𝐼4 = 𝐴
𝐼3 = 𝐴 7
2 7
𝐼1 = −2 − (− ) 12
3 𝐼5 = 𝐴
7
2
= −2 +
3
4
𝐼1 = − 𝐴
3
Eliminate 𝐼3 from equation 4

2𝐼3 − 𝐼4 = 0
𝐼4
𝐼3 = - substitute to equation 2
2
From equation 2

𝐼3 + 𝐼4 − 𝐼5 = 2
𝐼4
+ 𝐼4 − 𝐼5 = 2
2
1.5𝐼4 − 𝐼5 = 2 – equation 6
• In the DC circuit as shown, solve the node voltages and branch currents in the
given circuit using nodal analysis. Use Cramer’s Rule or Elimination.

KCL at Node 1

−𝐼1 − 𝐼4 − 𝐼2 − 2 = 0

𝐼1 + 𝐼2 + 𝐼4 = −2
𝑉1 − 𝑉2 𝑉1 − 𝑉0 𝑉1 − 10
{[ ]+[ ]+[ ] = 0}4
4 1 2
(𝑉1 − 𝑉2 ) + 4𝑉1 − 4𝑉0 + 2𝑉1 − 20 = 0

7𝑉1 − 𝑉2 = 20 – equation 1

KCL at Node 2

−𝐼1 − 𝐼3 + 2 + 𝐼1 = 0

𝐼3 = 2
𝑉2 −𝑉0
=2
2

𝑉2
=2
2
𝑉2 = 4𝑉

From equation 1 𝑉2 − 𝑉0
𝐼3 =
2
7𝑉1 − 𝑉2 = 20
4−0
=
7𝑉1 − 4 = 20 2

7𝑉1 = 20 + 4 𝐼3 = 2𝐴

7𝑉1 = 24 𝑉1 − 10
𝐼4 =
2
24
𝑉1 = 𝑉 24
7 − 10
= 7
𝑉1 − 𝑉2 2
𝐼1 =
4 𝐼4 = −3.2857𝐴
24
−4
= 7
4 24
𝑉1 = 𝑉
𝐼1 = −0.1429𝐴 7

𝑉1 − 𝑉0 𝑉2 = 4𝑉
𝐼2 =
1 𝐼1 = −0.1429𝐴
24
−0 24
= 7 𝐼2 = 𝐴
1 7
24 𝐼3 = 2𝐴
𝐼2 = 𝐴
7
𝐼4 = −3.2857𝐴
• Determine 𝑉0 in the given circuit below using the superposition principle.

Let 2A acts alone

𝑅 = 6Ω‖3Ω
6×3
= = 2Ω
6+3
𝑅 = 4Ω‖12Ω
4 × 12
= = 3Ω
4 + 12

Using Current Divider


𝑅2
𝑖= 𝐼
𝑅 = 2Ω ∫ 3Ω = 5Ω 𝑅2 + 𝑅1 𝑆
5
𝑖= 2 = 1𝐴
5+5

Using Ohm’s Law

𝑉0 1 = 𝐼𝑅

= 1(5)
𝑉0 1 = 5𝑉

Let 19V acts alone KCL at Node 𝑉1

𝐼1 −𝐼2 −𝐼3 = 0
𝑉0 − 𝑉1 𝑉1 − 𝑉0 𝑉1 − 𝑉2
( )−( )−( )=0
6 3 5
𝑉1 𝑉1 𝑉1 𝑉2
[− − − + = 0] 30
6 3 5 5
5𝑉1 − 10𝑉1 + 6𝑉1 − 6𝑉2 = 0

[21𝑉1 − 6𝑉2 = 0] ÷ 3

7𝑉1 − 2𝑉2 = 0 – equation 1


KCL at Node 𝑉2 Using Equation Mode in Calculator

𝐼3 −𝐼4 −𝐼5 = 0 𝑉1 = 2.85𝑉

𝑉1 − 𝑉2 𝑉2 − 𝑉0 𝑉2 − 19 𝑉2 = 9.975𝑉
( )−( )−( )=0
5 12 4
𝑉0 2 = 𝑉1 − 𝑉2
𝑉1 𝑉2 𝑉2 𝑉2 19
[ − − − + = 0] 60 = 2.85𝑉 − 9.975𝑉
5 5 12 4 4
12𝑉1 − 12𝑉2 − 5𝑉2 − 15𝑉2 + 285 = 0 𝑉0 2 = −7.125𝑉

12𝑉1 − 32𝑉2 = −285 – equation 2

Let 12V acts alone KCL at Node 𝑉1

𝐼1 −𝐼2 −𝐼3 = 0
12 − 𝑉1 𝑉1 − 𝑉0 𝑉1 − 𝑉2
( )−( )−( )=0
6 3 5
12 𝑉1 𝑉1 𝑉2
[ − − + = 0] 30
6 3 5 5
−60 + 5𝑉1 + 10𝑉1 + 6𝑉1 − 6𝑉2 = 0
KCL at Node 𝑉2
5𝑉1 + 10𝑉1 + 6𝑉1 − 6𝑉2 = 60
𝐼3 −𝐼4 −𝐼5 = 0
[21𝑉1 − 6𝑉2 = 60] ÷ 3
𝑉1 − 𝑉2 𝑉2 − 𝑉0 𝑉2 − 𝑉0
( )−( )−( )=0 7𝑉1 − 2𝑉2 = 20 – equation 1
5 12 4
𝑉1 𝑉2 𝑉2 𝑉2
[ − − − = 0] 60
5 5 12 4
12𝑉1 − 12𝑉2 − 5𝑉2 − 15𝑉2 = 0

[12𝑉1 − 32𝑉2 = 0] ÷ 4

3𝑉1 − 8𝑉2 = 0 – equation 2

Using Equation Mode in Calculator

𝑉1 = 3.2𝑉

𝑉2 = 1.2𝑉

𝑉0 3 = 𝑉1 − 𝑉2

= 3.2𝑉 − 1.2𝑉

𝑉0 3 = 2𝑉

𝑉0 = 𝑉0 1 + 𝑉0 2 + 𝑉0 3

= 5𝑉 + (−7.125𝑉) + 2𝑉

𝑉0 = −0.125𝑉
• For the circuit in figure below, use source transformation to find i

Transform 2A to a Voltage Source


𝑉 = 𝐼𝑅
= 2𝐴(5Ω)
𝑉 = 10𝑉

𝑅 = 5Ω ∫ 5Ω = 10Ω

Transform 10V and 20V to a Current Source


𝑉 𝑉
𝐼1 = 𝐼2 =
𝑅 𝑅
10𝑉 20𝑉
𝐼1 = = 1𝐴 𝐼2 = = 2𝐴
10Ω 10Ω

Parallel Connection
𝑅 = 10Ω‖10Ω 2𝐴 − 1𝐴 = 1𝐴
10 × 10
= = 5Ω
10 + 10
Using Current Divider
𝑅2
𝑖= 𝐼
𝑅2 + 𝑅1 𝑆
5Ω
𝑖= 1 = 0.5556𝐴
5Ω + 4Ω

• Find the Thevenin’s and Norton’s equivalent circuit. Also solve for the maximum
power transfer.

𝑅𝑇𝐻 = 10Ω‖40Ω + 20Ω


10Ω × 40Ω
𝑅𝑇𝐻 = + 20
10Ω + 40Ω
𝑅𝑇𝐻 = 28Ω
KCL at Node 𝑉1 Thevenin’s Equivalent Circuit
𝐼1 −𝐼2 −𝐼3 − 3 = 0
40 − 𝑉1 𝑉1 − 𝑉0 𝑉1 − 𝑉2
( )−( )−( )−3=0
10 40 20
𝑉1 𝑉1 𝑉1 𝑉2
[4 − − − + − 3 = 0] 30
10 40 20 20
40 − 4𝑉1 − 𝑉1 − 2𝑉1 + 2𝑉2 = 0

[21𝑉1 − 6𝑉2 = 0] ÷ 3

7𝑉1 − 2𝑉2 = 0 – equation

KCL at Node 𝑉𝑇𝐻 Norton’s Equivalent Circuit

3 + 𝐼3 = 0
𝑉1 − 𝑉𝑇𝐻
3+ =0
20
𝑉1 𝑉2
[3 + − = 0] 20
20 20
60 + 𝑉1 − 𝑉2 = 0

𝑉1 − 𝑉2 = −60 – equation 2

Using Equation Mode in Calculator

𝑉1 = 32𝑉

𝑉𝑇𝐻 = 92𝑉
𝑉𝑇𝐻
𝐼𝑁 =
𝑅𝑇𝐻
92𝑉
=
28Ω
23
𝐼𝑁 = 𝐴 = 3.2857𝐴
7
𝑅𝑇𝐻 = 𝑅𝑁 = 28Ω

𝑉𝑇𝐻 2
𝑃𝑚𝑎𝑥 =
4𝑅𝑇𝐻

922
=
4(28)
529
𝑃𝑚𝑎𝑥 = 𝑊 = 75.5714𝑊
7

You might also like